1
$\begingroup$

Can anyone provide the proof for the Karamata Inequality?

  • 1
    http://www.artofproblemsolving.com/Forum/viewtopic.php?highlight=majorization+karamata&t=149752012-01-08
  • 1
    http://en.wikipedia.org/wiki/Karamata%27s_inequality2012-01-08

0 Answers 0